Jump to content
BrainDen.com - Brain Teasers
  • 0


Guest
 Share

Question

Each of M and N is a positive integer. Determine all possible value(s) of a positive prime constant P such that the equation: 2M – 3*P = N2 has precisely two distinct solutions in (M, N).

Link to comment
Share on other sites

5 answers to this question

Recommended Posts

  • 0

Each of M and N is a positive integer. Determine all possible value(s) of a positive prime constant P such that the equation: 2M – 3*P = N2 has precisely two distinct solutions in (M, N).

There are none.

N can't be 0 mod 3 as that would imply that 2^M is divisible by 3. If N is 1 mod 3, then 3 must divide 2^M-1 and so M must be even (it is easy to show that 3 divides 2^X-1 iff X is even). If N is 2 mod 3, then 3 must divide 2^(M-2)-1 and again M must be even. So, M is even. Then, we can rearrange and factor to get: (2^(M/2)-N)*(2^(M/2)+N)=3*P So, either [(2^(M/2)-N)=P and (2^(M/2)+N)=3] or [(2^(M/2)-N)=3 and (2^(M/2)+N)=P]. In either case, setting P determines both N and M uniquely. Therefore, there is no P which will give two distinct solutions in (N,M).

Edited by superprismatic
Link to comment
Share on other sites

  • 0

There are none.

N can't be 0 mod 3 as that would imply that 2^M is divisible by 3. If N is 1 mod 3, then 3 must divide 2^M-1 and so M must be even (it is easy to show that 3 divides 2^X-1 iff X is even). If N is 2 mod 3, then 3 must divide 2^(M-2)-1 and again M must be even. So, M is even. Then, we can rearrange and factor to get: (2^(M/2)-N)*(2^(M/2)+N)=3*P So, either [(2^(M/2)-N)=P and (2^(M/2)+N)=3] or [(2^(M/2)-N)=3 and (2^(M/2)+N)=P]. In either case, setting P determines both N and M uniquely. Therefore, there is no P which will give two distinct solutions in (N,M).

I cannot refute the methodology corresponding to the foregoing proof. However, P=5 seems to satisfy all the conditions of the given problem due to the following reasons:

The equation becomes 2^M – 15 = N^2 at P = 5. Reducing this to mod 3, it can easily be shown that: M must be even, so that: (2^M/2 + N) (2^M/2 - N) = 15.

Since, M and N are positive integers it follows that: 2^M/2 + N > 2^M/2 – N, and accordingly:

(2^M/2 + N, 2^M/2 - N) = (15, 1), (5, 3), giving: (2^M/2, N) = (8, 7), (4, 1), so that:

(M, N) = (6, 7), (4, 1).

Consequently, it seems that there exists at least one value of P that satisfies all the given conditions.

Although I came across this problem in an old mathematics periodical, the said magazine does not have the answer to this problem.

Till date, I have not yet been able to come up with a proof giving all the value(s) of P and I'm still looking for the same.

Edited by K Sengupta
Link to comment
Share on other sites

  • 0

I cannot refute the methodology corresponding to the foregoing proof. However, P=5 seems to satisfy all the conditions of the given problem due to the following reasons:

The equation becomes 2^M – 15 = N^2 at P = 5. Reducing this to mod 3, it can easily be shown that: M must be even, so that: (2^M/2 + N) (2^M/2 - N) = 15.

Since, M and N are positive integers it follows that: 2^M/2 + N > 2^M/2 – N, and accordingly:

(2^M/2 + N, 2^M/2 - N) = (15, 1), (5, 3), giving: (2^M/2, N) = (8, 7), (4, 1), so that:

(M, N) = (6, 7), (4, 1).

Consequently, it seems that there exists at least one value of P that satisfies all the given conditions.

Although I came across this problem in an old mathematics periodical, the said magazine does not have the answer to this problem.

Till date, I have not yet been able to come up with a proof giving all the value(s) of P and I'm still looking for the same.

Thanks, you helped me find the kink in my reasoning!

Thanks, you have found the flaw in my argument! I didn't consider one other case, namely, that (2^(M/2)-N)=1 and (2^(M/2)+N)=3*P. So, when P=5, you get the (8,7) answer and from the (2^(M/2)-N)=3 and (2^(M/2)+N)=P possibility, you get the (4,1) answer. So, that means that this pair of possibilities are the only way you can get 2 answers (the other case, when [(2^(M/2)-N)=P and (2^(M/2)+N)=3], is actually inconsistent because P must be 2 -- the only prime less than 3). Anyway, in my overlooked case, (2^(M/2)-N)=1 and (2^(M/2)+N)=3*P, P must be of the form 1+2^2+2^4+2^6+...+2^(2n) for some n. 5 is the case when n=1. If there are any other primes of this form, then there are other answers with 2 distinct solutions in (M,N). I checked up to n=20 with no luck. If I find another, I'll be sure to let you know. Thanks for the wonderful problem and for giving me the hint to the flaw in my "proof"!

Link to comment
Share on other sites

  • 0

And it is now possible to determine that P can only be 5, which solves Sengupta's original puzzle.

We have already found from previous posts that, if

2^M - 3P = N^2, then M must be even so that, putting M = 2m

(2^m - N)(2^m + N) = 3P, an equation which can only be solved if

(a) 2^j - N = 3 and 2^j + N = P so that 2^(j+1) = P + 3

and/or

(b) 2^k - N = 1 and 2^k + N = 3P so that 2^(k+1) = 3P + 1.

If the problem as stated possesses two solutions for the same prime number P, then

3 * 2^(j+1) - 2^(k+1) = 8

i.e.

3 * 2^(j-2) - 2^(k-2) = 1.

Clearly the only solution to this Diophantine equation is j = 2 and k = 3.

This immediately yields P = 5.

Edited by jerbil
Link to comment
Share on other sites

  • 0

And it is now possible to determine that P can only be 5, which solves Sengupta's original puzzle.

We have already found from previous posts that, if

2^M - 3P = N^2, then M must be even so that, putting M = 2m

(2^m - N)(2^m + N) = 3P, an equation which can only be solved if

(a) 2^j - N = 3 and 2^j + N = P so that 2^(j+1) = P + 3

and/or

(b) 2^k - N = 1 and 2^k + N = 3P so that 2^(k+1) = 3P + 1.

If the problem as stated possesses two solutions for the same prime number P, then

3 * 2^(j+1) - 2^(k+1) = 8

i.e.

3 * 2^(j-2) - 2^(k-2) = 1.

Clearly the only solution to this Diophantine equation is j = 2 and k = 3.

This immediately yields P = 5.

Nice finish, Jerbil! Your completion also means (see my previous observations) that the only prime

of the form 1+sum{i=1 to n}(4^i) is 5. Now, we can have a real number theory type proof of this

fact starting with "For a given prime P, consider multiple solutions of 2^M - 3P = N^2 ....".

Link to comment
Share on other sites

Join the conversation

You can post now and register later. If you have an account, sign in now to post with your account.

Guest
Answer this question...

×   Pasted as rich text.   Paste as plain text instead

  Only 75 emoji are allowed.

×   Your link has been automatically embedded.   Display as a link instead

×   Your previous content has been restored.   Clear editor

×   You cannot paste images directly. Upload or insert images from URL.

Loading...
 Share

  • Recently Browsing   0 members

    • No registered users viewing this page.
×
×
  • Create New...